EXAM 1

अब Quizwiz के साथ अपने होमवर्क और परीक्षाओं को एस करें!

A nurse is caring for a client who requires a replacement peripheral IV. The client is dehydrated and requires a smaller gauge catheter than the #20-gauge being replaced. Which of the following gauge catheters should the nurse plan to use? #16-gauge #18-gauge #22-gauge #14-gauge

22

a "blue"(cyanotic) client needs

RAPID response team. Resp rate of 6 is BAD. Waiting 15 minutes would probably kill the client.

A nurse is preparing to administer ceftriaxone 1 g in 100 mL IV over 30 min. The drip rate is 10 gtt/mL. The nurse should set the infusion rate to administer how many gtt/min? (Round the answer to the nearest whole number. Use a leading zero if it applies. Do not use a trailing zero.) gtt/min

To determine the correct flow rate, divide the volume to be infused by the time in minutes and multiply by the drop factor. So, 100 mL (volume to be infused) divided by 30 min (time in min) x 10 gtt/mL (drops per mL) = 33 gtt/min.

When should the transfusion be started after obtaining the blood?

You should begin the transfusion within 30 minutes of obtaining the blood on your unit, so it is important to be prepared prior to releasing it from the blood bank

According to the ATI video, what should the nurse do immediately prior to starting a blood transfusion? a. Attach the distal end of the administration set to the IV catheter. b. Remove the product identification document from the blood product. c. Prime the tubing with D5W. d. Obtain and record the client's vital signs.

obtain and record the client's vital signs.

The nurse has gathered the correct blood administration set. Which type of intravenous solution will the nurse use to prime this tubing? a. lactated ringer b. 5% dextrose c.0.45% sodium cl d.0.9% sodium chloride

0.9% sodium chloride

A nurse is assessing a 4-year-old child who has severe dehydration. Which of the following manifestations should the nurse expect? 10% weight loss Respiratory rate of 18/min Capillary refill 3 seconds Urine output 24 mL/hr

10% weight lossMY ANSWERThe nurse should recognize that a child who has severe dehydration will exhibit a weight loss of 10% or greater Respiratory rate of 18/minThe nurse should recognize that a child who has severe dehydration will exhibit hyperpnea, which is deep and rapid respiration. Capillary refill 3 secondsThe nurse should recognize that a child who has severe dehydration will exhibit a delayed capillary refill of greater than 4 seconds. Urine output 24 mL/hrThe nurse should recognize that a urine output of 24 mL/hr is within the expected reference range for a 4-year-old child. A child who has severe dehydration will exhibit oliguria or anuria.

A nurse is caring for a client who is receiving a blood transfusion and reports itching. The nurse observes areas of urticaria on the client's skin. Which of the following actions should the nurse take? Administer prescribed antipyretic. Stop the blood transfusion.Reevaluate the client in 15 min.Apply a warm compress to the affected areas.

Administer prescribed antipyretic.The nurse would administer a prescribed antihistamine for this client, not an antipyretic. Stop the blood transfusion.This client is exhibiting manifestations of a mild allergic reaction to the blood transfusion. The nurse should stop the transfusion and follow facility protocol regarding transfusion reactions. Reevaluate the client in 15 min.MY ANSWERDelaying intervention could potentially lead to poor outcomes for the client. While itching and urticaria are signs of a mild allergic reaction, they might also be signs of more serious transfusion reactions such as anaphylaxis. Apply a warm compress to the affected areas.To relieve the itching associated with urticaria, the nurse should apply a cool compress to the area.

A nurse is preparing to start an IV for a client who has a high risk for bleeding. Which of the following actions should the nurse take? Apply a cold compress to the selected IV site. Ask the client to hold the extremity up prior to searching for an IV site. Ask the client to spread the fingers of the selected extremity. Apply a blood pressure cuff set to 30 mm Hg.

Apply a cold compress to the selected IV site.The nurse should apply a warm compress to the selected IV site. Applying a cold compress to the site could compromise the client's circulation. Ask the client to hold the extremity up prior to searching for an IV site.The nurse should ask the client to hold the extremity in a dependent position to enhance blood flow to the site. Ask the client to spread the fingers of the selected extremity.The nurse should ask the client to make a fist with the fingers of the selected extremity. The client should then open and close their fist several times to enhance blood flow to the area. Apply a blood pressure cuff set to 30 mm Hg.MY ANSWERInstead of using a tourniquet, the nurse should apply a blood pressure cuff set to 30 mm Hg prior to starting an IV for this client. This will help protect the client's extremity from bruising and bleeding.

Put the following pre-procedure steps in the correct order to be done prior to administering blood to a client. Identify client by checking ID band and asking their name and DOB. Check MD order. Introduce yourself. Verify client allergies Gather supplies and equipment.

MD order supplies introduce self id patient verify allergies

A nurse is caring for a client who has a peripherally inserted central catheter (PICC) line. The nurse should identify that which of the following information is true about this type of IV route? A PICC line is a midline catheter used to administer blood. A PICC line is a catheter that allows for infusion of IV fluids without an infusion pump. A PICC line is a long catheter inserted through the veins of the antecubital fossa. A PICC line is a catheter that is used for emergent or trauma situations.

PICC line is a midline catheter used to administer blood.A PICC line is a central catheter that is inserted in the cephalic vein, and the tip of the catheter rests in the superior vena cava of the heart. A PICC line is a catheter that allows for infusion of IV fluids without an infusion pump.An infusion pump is needed for a PICC line. A PICC line is a long catheter inserted through the veins of the antecubital fossa.MY ANSWERPICC lines have lower complication rates because they are inserted in the upper extremity. A PICC line is a catheter that is used for emergent or trauma situations.A nontunneled central catheter is used for emergent or trauma situations, not a PICC line.

A client with a history of congestive heart failure is receiving a blood transfusion. During the transfusion the client starts experiencing the following signs and symptoms: persistent cough, crackles, hypertension, and distended neck veins. Based on these assessment findings which intervention should the nurse implement? a. Administer antibiotics as prescribed. b. Administer an antihistamine as prescribed. c. Send a urine sample to the lab. d. Place the client in an upright position.

Place the client in an upright position

A nurse is assessing a client who has acute alcohol intoxication. The nurse should identify that the client is at risk for which of the following acid-base imbalances? Respiratory acidosis Respiratory alkalosis Metabolic acidosis Metabolic alkalosis

Respiratory acidosiSCommon causes of respiratory acidosis include electrolyte imbalances, inadequate chest expansion, respiratory depression, and airway obstruction. Respiratory alkalosisCommon causes of respiratory alkalosis include mechanical ventilation, aspirin toxicity, shock, anxiety, and hyperventilation. Metabolic acidosisCommon causes of metabolic acidosis include alcohol or ethanol intoxication, diabetic ketoacidosis, hypoxia, kidney failure, diarrhea, and pancreatitis. C;Metabolic alkalosisCommon causes of metabolic alkalosis include total parenteral nutrition, blood transfusions, nasogastric suctioning, thiazide diuretics, and hypercortisolism.

A nurse is caring for a client who has chronic kidney disease (CKD) and has developed Kussmaul respirations. The nurse should identify that the client is experiencing which of the following acid-base imbalances? Respiratory acidosis Respiratory alkalosis Metabolic acidosis Metabolic alkalosis

Respiratory acidosisA client experiencing respiratory acidosis will manifest an increase in the respiratory rate, but not in depth as with Kussmaul respirations. Respiratory alkalosis A client experiencing respiratory alkalosis will report shortness of breath and chest tightness. C; Metabolic acidosisAcid retention increases with advancing CKD. A client develops Kussmaul respirations (increase in depth and rate) to promote excretion of carbon dioxide through the lungs causing metabolic acidosis. Metabolic alkalosisA client experiencing metabolic alkalosis will manifest a decrease in the rate and depth of respirations.

How fast do you start the blood? Explain your answer?

Start infusing the blood slowly; each facility policy will be different, but the typical rates are less than 5 mL/minute. Faster infusion rates can lead to damaged or unsafe blood. Faster infusion would also increase consequences of a negative reaction. If no problems are noted during the first 15 minutes of transfusion, then you can document findings, increase the infusion rate, and decrease observation according to your facility's policy. Ideally PRBCs should infuse over 2.5 to 3 hours. To avoid risk of bacterial growth, PRBCs should not hang for more than 4 hours

The client receiving a unit of PRBCs (packed red blood cells) develops wheezing, itching, hives, and becomes flushed. Which intervention should the nurse implement first? a. Slow the rate of the transfusion. b. Treat the symptoms. c. Stop the transfusion. d. Notify the healthcare provider.

Stop the transfusion.

The nurse understands an example of a systemic intravenous complication is Septicemia. T/F

True

A nurse is providing teaching to a client who has chronic kidney failure (CKF) and is scheduled to begin hemodialysis treatments. Which of the following client statements indicates an understanding of the teaching? "I will be sure to weigh myself every week when I start dialysis treatments." "I will be on dialysis treatments until I can start urinating on my own again." "I will only need dialysis treatments when my lab values are abnormal." "I will have to have dialysis treatments two or three times a week."

"I will be sure to weigh myself every week when I start dialysis treatments."Clients who are undergoing hemodialysis treatment should be taught to weigh themselves every day. "I will be on dialysis treatments until I can start urinating on my own again."Clients who have CKF that requires dialysis have reached a stage of their disease in which renal replacement or transplantation is required. Their kidneys will not recover to provide for adequate urination. "I will only need dialysis treatments when my lab values are abnormal."Once renal replacement therapy starts, it will have to be continued on a routine basis of two to three treatments per week, unless a kidney transplant is performed. "I will have to have dialysis treatments two or three times a week."MY ANSWERHemodialysis treatments vary with each client, but are usually performed two to three times per week.

A nurse is providing teaching to a client who has chronic kidney failure (CKF) and is scheduled to begin hemodialysis. Which of the following statements should the nurse make about the hemodialysis procedure? "Toxins, waste, and excess fluids are removed from arterial blood using a machine two to three times per week." "Waste, fluid, and electrolytes are exchanged within the peritoneal cavity by gravity 7 days per week." "A tissue compatible kidney is received from a living or cadaver donor." "Waste, fluid, and electrolytes are exchanged within the peritoneal cavity using a cycling machine during sleeping hours."

"Toxins, waste, and excess fluids are removed from arterial blood using a machine two to three times per week."MY ANSWERThe nurse should instruct the client that hemodialysis is a process that removes toxins and waste from the blood and eliminates excess fluids. Hemodialysis involves the use of vascular access, a dialyzer, dialysate, and a hemodialysis machine that regulates and monitors the process. "Waste, fluid, and electrolytes are exchanged within the peritoneal cavity by gravity 7 days per week."The nurse should use this description to describe continuous ambulatory peritoneal dialysis. "A tissue compatible kidney is received from a living or cadaver donor."The nurse should use this description to describe a kidney transplant. "Waste, fluid, and electrolytes are exchanged within the peritoneal cavity using a cycling machine during sleeping hours."The nurse should use this description to describe automated peritoneal dialysis.

The client has an order for IV Lasix how will you give it?

- Due to all medications being incompatible with blood products, medications should never be pushed through the I.V. line with the blood as it can cause hemolysis. Given a patient has an order for IV Lasix, the blood transfusion needs to be interrupted to administer it safely. Once blood transfusion is paused, the nurse would flush the line with at least 20 mL of 0.9% sodium chloride solution, give the medication, flush again with an additional 20 mL or more of 0.9% sodium chloride solution, and then restart the blood transfusion.

Why do clients receive blood transfusions?

- Patients receive blood and blood products for various reasons, s uch as anemia or blood loss due to trauma, surgery, or certain medical conditions

What type of IVF is administered with blood transfusions?

0.9% normal saline solution is the only fluid compatible with blood transfusions and should be connected via a Y-type blood administration at typically 250 or 500 mL, varying per facility policy

A nurse is calculating a client's intake and output for the last 4 hr. The client consumed 480 mL of water and 240 mL of coffee. The client has also received IV fluids for 4 hr infusing at 100 mL/hr. Which of the following amounts represents the client's intake over the last 4 hr? 1,120 mL 720 mL 480 mL 580 mL

1,120 mL

A nurse is preparing to administer 4,200 mL of intravenous fluids to a client to infuse over 24 hr. The nurse should set the IV pump to deliver how many mL/hr? (Round the answer to the nearest whole number. Use a leading zero if it applies. Do not use a trailing zero.) mL/hr

1ollow these steps to calculate the infusion rate using the Ratio and Proportion or Desired Over Have method of calculation: Step 1: What is the unit of measurement the nurse should calculate? mL/hr Step 2: What is the volume the nurse should infuse? 4200 mL Step 3: What is the total infusion time? 24 hr Step 4: Should the nurse convert the units of measurement? No Step 5: Set up an equation and solve for X. Volume (mL)X mL/hr = Time (hr) 4200 mLX mL/hr = 24 hr X mL/hr = 175 mL/hr Step 6: Round if necessary. Step 7: Determine if the amount to administer makes sense. If the prescription reads 4200 mL of fluids IV to infuse over 24 hr, it makes sense to administer 175 mL/hr. The nurse should set the IV pump to deliver 4200 mL of fluids IV at 175 mL/hr. Follow these steps to calculate the infusion rate using the Dimensional Analysis method of calculation: Step 1: What is the unit of measurement the nurse should calculate? (Place the unit of measure being calculated on the left side of the equation.) X mL/hr = Step 2: Determine the ratio that contains the same unit as the unit being calculated. (Place the ratio on the right side of the equation, ensuring that the unit in the numerator matches the unit being calculated.) 4200 mLX mL/hr = 24 hr Step 3: Place any remaining ratios that are relevant to the item on the right side of the equation, along with any needed conversion factors, to cancel out unwanted units of measurement. 4200 mLX mL/hr = 24 hr Step 4: Solve for X. X mL/hr = 175 mL/hr Step 5: Round if necessary. Step 6: Determine if the amount to administer makes sense. If the prescription reads 4200 mL of fluids IV to infuse over 24 hr, it makes sense to administer 175 mL/hr. The nurse should set the IV pump to deliver 4200 mL of fluids IV at 175 mL/hr.75

A nurse is reviewing prescriptions for a client who needs intravenous fluid replacement therapy due to vomiting and diarrhea. Which of the following fluid prescriptions should the nurse expect to initiate? 3% sodium chloride solution 0.9% sodium chloride solution 0.45% sodium chloride solution Dextrose 10% in water

3% sodium chloride solutionA 3% sodium chloride solution is hypertonic and is used for emergent replacement of solutes, such as in clients experiencing manifestations of hyponatremia. This solution can cause fluid shifts and is not the best option for a client experiencing vomiting and diarrhea. 0.9% sodium chloride solutionMY ANSWERA 0.9% sodium chloride solution is isotonic and is used for hydration needs such as from vomiting, diarrhea, hemorrhage, and shock. This is the most appropriate solution for the provider to prescribe for this client. 0.45% sodium chloride solutionA 0.45% sodium chloride solution is hypotonic and is used to treat hypernatremia and diabetic ketoacidosis. This solution is not the best option for a client experiencing vomiting and diarrhea. Dextrose 10% in waterDextrose 10% in water is hypertonic and is used to treat hypoglycemia. This solution is not the best option for a client experiencing vomiting and diarrhea.

A nurse is caring for a client who has heart failure and a prescription to receive a unit of packed red blood cells. The nurse should plan to infuse the blood over which of the following lengths of time? 1 hr 2 hr 4 hr 6 hr

4 Blood can be administered over a period of 1 to 4 hr. For a client who is at risk for circulatory overload, such as a client who has heart failure, a disorder in which compromised cardiac output results in poor tissue perfusion and fluid overload, the transfusion should be administered slowly (maximum time of 4 hr) to avoid increasing the workload of the heart.

A nurse is receiving report on four clients. The nurse should identify that which of the following clients might be experiencing hypomagnesemia? A client who has vomited four times during the last 8 hr. A client who requested an extra breakfast tray to eat. A client who can ambulate without assistance. A client who reports extreme thirst.

A client who has vomited four times during the last 8 hr.MY ANSWERNausea and vomiting are early manifestations of hypomagnesemia. A client who requested an extra breakfast tray to eat.Requesting an extra tray to eat can indicate an increased appetite. The nurse should identify that a decreased appetite, rather than an increased appetite, is a manifestation of hypomagnesemia. A client who can ambulate without assistance.A client who has hypomagnesemia might experience weakness, which could make it difficult to ambulate. Therefore, the nurse should identify that difficulty ambulating, rather than the ability to ambulate without assistance, could be an indication of hypomagnesemia. A client who reports extreme thirst.Extreme thirst is not a manifestation of hypomagnesemia. A client who is experiencing extreme thirst might be experiencing hypernatremia.

A nurse is caring for a client who has chronic kidney disease (CKD). The client suddenly develops restlessness and dyspnea, and the nurse auscultates crackles in the client's lungs. Which of the following actions should the nurse take first? Administer IV furosemide. Obtain an oxygen saturation level. Administer IV morphine sulfate. Place the client in a high-Fowler's position.

Administer IV furosemide.The nurse should administer IV furosemide to the client experiencing pulmonary edema to promote diuresis. However, there is another action the nurse should take first. Obtain an oxygen saturation level.The nurse should obtain an oxygen saturation level on the client experiencing pulmonary edema to determine effectiveness of oxygen delivery However, there is another action the nurse should take first. Administer IV morphine sulfate.The nurse should administer IV morphine sulfate to the client experiencing pulmonary edema to reduce oxygen demand. However, there is another action the nurse should take first. Place the client in a high-Fowler's position.MY ANSWERThe first action the nurse should take when using the airway, breathing, circulation approach to client care is to place the client experiencing pulmonary edema in a high-Fowler's position. This action, along with the application of oxygen, facilitates gas exchange and increases the ease of breathing.

A nurse is monitoring a client who has received enalapril for heart failure. The nurse should identify that which of the following findings as an adverse effect of this medication? Bradycardia Orthostatic hypotension Increased perspiration Somnolence

Bradycardia Tachycardia is an adverse effect of enalapril. C; Orthostatic hypotensionEnalapril, an angiotensin-converting enzyme, prevents conversion of angiotensin I to angiotensin II, which causes dilation of the arterial and venous vessels. This can cause an adverse effect of orthostatic hypotension. Increased perspirationDecreased perspiration is an adverse effect of enalapril. Clients who are taking enalapril should avoid becoming overheated in warmer weather. SomnolenceInsomnia is an adverse effect of enalapril.

The client complains to the nurse, "Where the nurse took out my IV looks like it is bruised and I feel a lump there. It is really uncomfortable too." The nurse recognizes this as which of the following? a. Local infection b. Hematoma c. Infiltration d. Phlebitis

Hematoma

Which of the following is the correct rational for infusing a unit of blood in less than 4 hours? a. The blood has the potential for bacterial growth if allowed to infuse longer. b. The blood will coagulate if left out of the refrigerator for greater than 4 hours. c. The blood components begin to break down after 4 hours. d. The blood will not be affected; this is a laboratory procedure.

The blood has the potential for bacterial growth if allowed to infuse longer.

The nurse is getting ready to administer blood to a client. What should the nurse include in the pre-procedure steps? Select all that apply. a. Verify client allergies. b.Gather supplies and equipment. c. Check MD order. d. Identify the client by checking the ID band and asking their name and DOB. e. Introduce yourself.

a. Verify client allergies. b.Gather supplies and equipment. c. Check MD order. d. Identify the client by checking the ID band and asking their name and DOB. e. Introduce yourself.

hypokalemia it will affect

skeletal muscles, follow ABC's, assess respiratory status first. Also with hypokalemia caused by nasogastric suctioning (hyperkalemia is related to burns see PP slide 44)

A nurse administers the first dose of a client's prescribed antibiotic via intermittent IV bolus. During the first 10 to 15 min of administration, which of the following assessments is the nurse's priority? Assess the IV site for redness or swelling. Assess the client for a systemic allergic reaction. Assess the IV dressing for signs of leakage. Assess the client's limb for signs of discomfort.

ssess the IV site for redness or swelling.The nurse should assess the IV site for redness and swelling to determine phlebitis, which does not typically occur within the first 10 to 15 min of administration. However, there is another assessment that is the nurse's priority. Assess the client for a systemic allergic reaction.MY ANSWERThe greatest risk to this client is anaphylaxis. Therefore, the priority assessment is to assess the client for a systemic allergic reaction. Clients can experience a systemic allergic reaction rapidly with IV antibiotics and should be observed for the first 10 to 15 min for manifestations. Assess the IV dressing for signs of leakage.The nurse should assess the client's IV dressing for signs of leakage to determine if there is an infiltration. However, there is assessment that is the nurse's priority. Assess the client's limb for signs of discomfort.The nurse should assess the client's limb for signs of discomfort when receiving IV medication, which can be expected at first. However, there is another assessment that is the nurse's priority.

The nurse is getting ready to administer a unit of PRBCs (packed red blood cells) to a client with anemia. Which tubing will the nurse use to administer the blood? a. normal saline drip b. c. y tubin d. PB

y tubing

A nurse is reviewing a client's laboratory results. Which of the following results should the nurse report to the provider? Potassium 4.5 mEq/L Sodium 138 mEq/L Magnesium 3 mEq/L Calcium 10 mg/dL

A potassium level of 4.5 mEq/L is within the expected reference range of 3.5 to 5 mEq/L. Sodium 138 mEq/LA sodium level of 138 mEq/L is within the expected reference range of 135 to 145 mEq/L. answer: Magnesium 3 mEq/LA magnesium level of 3 mEq/L is above the expected reference range of 1.3 to 2.1 mEq/L. Therefore, the nurse should report this finding to the provider. Calcium 10 mg/dLA calcium level of 10 mg/dL is within the expected reference range of 9 to 10.5 mg/dL.

A nurse is caring for a client who has respiratory acidosis. Which of the following medications should the nurse prepare to administer? Antiemetic Hypoglycemic Antidiarrheal

AntiemeticThe nurse should administer an antiemetic medication to clients who have metabolic alkalosis caused by prolonged vomiting. HypoglycemicThe nurse should administer a hypoglycemic medication to clients who have metabolic acidosis caused by diabetic ketoacidosis. AntidiarrhealThe nurse should administer an antidiarrheal medication to clients who have metabolic acidosis caused by prolonged diarrhea. BronchodilatorMY ANSWERClients who have respiratory acidosis require interventions that improve oxygenation and ventilation and help maintain airway patency. Therefore, the nurse should prepare to administer oxygen, a bronchodilator, and possibly a mucolytic and an anti-inflammatory medication to a client who has respiratory acidosis.

A nurse is caring for a client who has a central venous access device in place. Which of the following routine interventions should the nurse use to prevent lumen occlusion? Apply a skin securement device to the catheter. Remove the dressing from the insertion site slowly and carefully. Use a pulsatile action while flushing. Have the client lie flat when changing administration sets or injection caps.

Apply a skin securement device to the catheter.This measure is intended to prevent catheter migration, not lumen occlusion. Remove the dressing from the insertion site slowly and carefully.tis measure is intended to prevent catheter dislodgement, not lumen occlusion. correct; Use a pulsatile action while flushing.Using a pulsatile action technique while flushing assists with the prevention of occlusion by removing possible solid deposits within the lumen. Have the client lie flat when changing administration sets or injection caps.This measure is intended to prevent air embolism, not lumen occlusion.

A nurse is reviewing the laboratory values for a client who is taking an ACE inhibitor and spironolactone for heart failure. The nurse should notify the provider for which of the following laboratory values? Creatinine 1.8 mg/dL Potassium 4.2 mEq/L BNP 98 pg/mL d-Dimer 0.3 mcg/mL

C; Creatinine 1.8 mg/dLThe nurse should notify the provider if the client has a creatinine of 1.8 mg/dL. Clients who are receiving an ACE inhibitor and spironolactone together can develop hyperkalemia and dysthymias. Potassium 4.2 mEq/LA potassium level of 4.2 mEq/L is within the expected reference range. However, the nurse should monitor for hyperkalemia when the client is receiving an ACE inhibitor and spironolactone together. BNP 98 pg/mL A BNP of 98 pg/mL is within the expected reference range. However, a BNP of greater than 100 pg/mL is a critical value that indicates ventricular stretching, which could lead to congestive heart failure. d-Dimer 0.3 mcg/mLA d-Dimer of 0.3 mcg/mL is within the expected reference range. However, a d-Dimer of greater than 0.4 mcg/mL can indicate formation of a thrombus, which could lead to a pulmonary embolus.

A nurse is reviewing the medical record of a client who has chronic kidney failure (CKF). Which of the following diseases in the client's history is a main cause of CKF? Diabetes mellitus Rheumatoid arthritis Asthma Gout

Diabetes mellitusMY ANSWERDiabetes mellitus and hypertension are the most common causes of CKF. Rheumatoid arthritisRheumatoid arthritis is not a known cause of CKF. AsthmaAsthma is not a known cause of CKF. GoutAlthough gout can lead to CKF, it is not a main cause. Diabetes and hypertension are the most common causes of CKF.

A nurse is preparing to obtain a blood sample from a client who has a triple-lumen central catheter in place. Which of the following actions should the nurse take? Discard the first 35 mL of aspirated blood before collecting the sample. Place the client in Trendelenburg position while withdrawing the blood sample. Withdraw the blood sample from the lumen that has the smallest diameter. Turn off the distal infusions for 1 to 5 min before obtaining the blood sample.

Discard the first 35 mL of aspirated blood before collecting the sample.The amount of aspirate to discard before collecting the sample varies with facility policy but is generally no more than 10 mL and often as little as 3 mL. Place the client in Trendelenburg position while withdrawing the blood sample.No specific client position is required for withdrawing blood through a central line. Withdraw the blood sample from the lumen that has the smallest diameter.The lumen with the largest diameter is generally the best choice for blood sampling. correct; Turn off the distal infusions for 1 to 5 min before obtaining the blood sample.To help ensure that the laboratory results will not be altered by the solutions infusing through the central access device, it is recommended that the nurse stop the distal infusions and clamp the tubing for 1 to 5 min before obtaining the blood sample. How long to stop the infusion varies with the type of infusion.

A 24 y.o. client needs a blood transfusion after surgery. What is an essential pre-procedure step the nurse must perform prior to administering blood? a. Complete the procedure in a timely manner. b. Ensure informed consent has been obtained. c. Document the date and time the transfusion started. d. Insert a 24 guage intravenous catheter.

Ensure informed consent has been obtained.

A nurse is caring for a client who has chronic kidney disease (CKD). The nurse should monitor the client for which of the following manifestations of fluid overload?

Flat neck veinsA client who is experiencing fluid overload due to CKD will manifest distended neck veins. Increased blood pressureMY ANSWERThe nurse should monitor the blood pressure of a client who has CKD. A client who is experiencing fluid overload due to CKD will experience an increase in blood pressure. Weak pulseA client who is experiencing fluid overload due to CKD will manifest a full, bounding pulse. Increased hematocritA client who is experiencing overhydration due to CKD will manifest a decreased hematocrit.

A nurse is preparing to administer 1,950 mL of 0.45% sodium chloride IV to infuse over 13 hr. The nurse should set the IV pump to deliver how many mL/hr? (Round the answer to the nearest whole number. Use a leading zero if it applies. Do not use a trailing zero.) mL/hr

Follow these steps to calculate the infusion rate using the Ratio and Proportion or Desired Over Have method of calculation: Step 1: What is the unit of measurement the nurse should calculate? mL/hr Step 2: What is the volume the nurse should infuse? 1950 mL Step 3: What is the total infusion time? 13 hr Step 4: Should the nurse convert the units of measurement? No Step 5: Set up an equation and solve for X. Volume (mL)X mL/hr = Time (hr) 1950 mLX mL/hr = 13 hr X mL/hr = 150 mL/hr Step 6: Round if necessary. Step 7: Determine if the amount to administer makes sense. If the prescription reads 1950 mL 0.45% sodium chloride IV to infuse over 13 hr, it makes sense to administer 150 mL/hr. The nurse should set the IV pump to deliver 0.45% sodium chloride IV at 150 mL/hr. Follow these steps to calculate the infusion rate using the Dimensional Analysis method of calculation: Step 1: What is the unit of measurement the nurse should calculate? (Place the unit of measure being calculated on the left side of the equation.) X mL/hr = Step 2: Determine the ratio that contains the same unit as the unit being calculated. (Place the ratio on the right side of the equation, ensuring that the unit in the numerator matches the unit being calculated.) 1950 mLX mL/hr = 13 hr Step 3: Place any remaining ratios that are relevant to the item on the right side of the equation, along with any needed conversion factors, to cancel out unwanted units of measurement. 1950 mLX mL/hr = 13 hr Step 4: Solve for X. X mL/hr = 150 mL/hr Step 5: Round if necessary. Step 6: Determine if the amount to administer makes sense. If the prescription reads 1950 mL 0.45% sodium chloride IV to infuse over 13 hr, it makes sense to administer 150 mL/hr. The nurse should set the IV pump to deliver 0.45% sodium chloride IV at 150 mL/hr.

A nurse is reviewing arterial blood gas (ABG) values for a client who is experiencing uncompensated metabolic acidosis. Which of the following ABG values should the nurse expect? (Select all that apply.) HCO3- 19 mEq/L pH 7.29 PaCO2 49 mm Hg pH 7.49 PaCO2 35 mm Hg

HCO3- 19 mEq/L is correct. The nurse should expect a client who is experiencing metabolic acidosis to have an HCO3- level below the expected reference range of 21 to 28 mEq/L, such as an HCO3- level of 19 mEq/L.pH 7.29 is correct. The nurse should expect a client who is experiencing metabolic acidosis to have pH level below the expected reference range of 7.35 to 7.45, such as a pH level of 7.29.PaCO2 49 mm Hg is incorrect. The nurse should expect a client who is experiencing metabolic acidosis to have a PaCO2 level within the expected reference range of 35 to 45 mm Hg. An elevated PaCO2 level of 49 mm Hg indicates respiratory acidosis.pH 7.49 is incorrect. Although pH 7.49 is above the expected reference range of 7.35 to 7.45, this is not an indication of metabolic acidosis. A pH value above the expected reference range indicates a state of alkalosis.PaCO2 35 mm Hg is correct. The nurse should expect a client who is experiencing metabolic acidosis to have a PaCO2 level within the expected reference range of 35 to 45 mm Hg, such as a PaCO2 level of 35 mm Hg.

A nurse is caring for a client who has an acid-base imbalance and is experiencing hypoxia. Which of the following actions should the nurse take first?Initiate continuous cardiac monitoring.Elevate the head of the client's bed.Instruct the client to deep breathe and cough.Initiate continuous SpO2 monitoring.

Initiate continuous cardiac monitoring.The nurse should initiate continuous cardiac monitoring for a client who is experiencing hypoxia to ensure the client does not develop dysrhythmias. However, there is another action the nurse should take first. Elevate the head of the client's bed.MY ANSWERThe first action the nurse should take when using the airway, breathing, circulation approach to client care is to elevate the head of the client's bed. Placing the client in the Fowler's or semi-Fowler's position will promote effective breathing and chest expansion. Instruct the client to deep breathe and cough.The nurse should instruct the client to deep breathe and cough to clear their airway. However, there is another action the nurse should take first. Initiate continuous SpO2 monitoring.The nurse should initiate continuous SpO2 monitoring for a client who is experiencing hypoxia to ensure the client does not decompensate. However, there is another action the nurse should take first.

A nurse is providing education to a client who has a prescription for a blood transfusion. Which of the following statements should the nurse include in the teaching? "I will check your vital signs every 15 minutes throughout the blood transfusion." "I might have a nursing assistant check on you periodically during the transfusion." "If you have no adverse effects in the first 15 to 30 minutes, you will not have any adverse effects later." "You must immediately report any symptoms like chills, nausea, or itching."

I will check your vital signs every 15 minutes throughout the blood transfusion."In most cases, the nurse will check the client's vital signs before the transfusion to establish a baseline, then 15 min after initiating the transfusion. After that, if the client shows no signs of a reaction, the nurse will check their vital signs every 30 min to 1 hr, depending on the client's overall condition, until 1 hr following the transfusion. "I might have a nursing assistant check on you periodically during the transfusion."Due to the complexity of the procedure and the life-threatening complications that might occur, monitoring a client during a blood transfusion is not something a nurse can delegate to an assistive personnel. It is acceptable for a nursing assistant to check on the client as long as the nurse is also monitoring the client regularly during the procedure. "If you have no adverse effects in the first 15 to 30 minutes, you will not have any adverse effects later."Serious reactions to transfusions typically occur early in the transfusion; however, there are other types of reactions that can occur later. Delayed hemolytic reactions, for example, can appear up to 14 days following transfusion, after the antibody level has increased enough to cause a reaction. Disease transmission from the blood product, although rare, is another example of a delayed reaction to a blood transfusion. "You must immediately report any symptoms like chills, nausea, or itching."MY ANSWERAlthough the nurse can identify objective signs of a transfusion reaction (changes in vital signs, flushing, cyanosis, coughing, and to some extent, dyspnea), the nurse might not be able to tell if the client is experiencing subjective symptoms (chills, nausea, chest pain, headache, backache, muscle pain). Subjective signs are important clues, and the nurse must be aware of them.

List 4 symptoms of an acute transfusion reaction, the cause and possible ways to prevent:

Itching, rash, chills and/or hives are amongst the various symptoms that can arise from an acute transfusion reaction. These effects shown by clients could be caused by an allergic reaction and/or a mild-to-moderate hypersensitivity due to antibodies in the donor blood. Preventive measures include administering antihistamines as prescribed to combat reactions, usually pretransfusion or between units. Patient awareness and education can also be a key component for quick medical action if reactions occur; therefore, instructing the patient and family on possible problems and encouraging them to notify you of any unusual changes during the transfusion is very important

Metabolic acidosis; will see __

Kussmaul respirations; resp system trying to blow off excess H+ ions to try to compensate for decreased pH by stimulating respirations

A nurse is caring for a client who has the following arterial blood gas (ABG) values: pH 7.44, PaCO2 37 mm Hg, and HCO3- 24 mEq/L. The nurse should identify that these values are an indication of which of the following? Metabolic acidosis Respiratory acidosis Acid-base balance Respiratory alkalosis

Metabolic acidosisMetabolic acidosis is indicated by a pH value less than 7.35 and an HCO3- value less than 21 mEq/L. Respiratory acidosisRespiratory acidosis is indicated by a pH value less than 7.35 and a PaCO2 value greater than 45 mm Hg. Acid-base balanceMY ANSWERThe nurse should identify that this client's ABG values are within the expected reference ranges. The expected reference ranges are: pH 7.35 to 7.45, PaCO2 35 to 45 mm Hg, and HCO3- 21 to 28 mEq/L. Respiratory alkalosisRespiratory alkalosis is indicated by a pH value greater than 7.45 and a PaCO2 value less than 35 mm Hg.

A nurse is caring for a client whose ABG results are pH 7.30, PaCO2 32 mm Hg, and HCO3- 19 mEq/L. The nurse should identify that the client has which of the following acid-base imbalances? Respiratory acidosis Respiratory alkalosis Metabolic acidosis Metabolic alkalosis

Respiratory acidosisFor a client who has respiratory acidosis, the nurse should expect low pH, high PaCO2, and the bicarbonate to be high or within the expected reference range. Respiratory alkalosisFor a client who has respiratory alkalosis, the nurse should expect high pH, low PaCO2, and the bicarbonate to be low or within the expected reference range. Metabolic acidosisMY ANSWERFor a client who has metabolic acidosis, the nurse should expect low pH, the PaCO2 to be low or within the expected reference range, and low bicarbonate. Metabolic alkalosisFor a client who has metabolic alkalosis, the nurse should expect high pH, the PaCO2 to be high or within the expected reference range, and high bicarbonate.

The student nurse is assessing the IV insertion site of the client. The student nurse recognizes the site is infiltrated. Which of the following signs and symptoms would be noted? a. The client complains of acute sharp pain and "pins & needles" feeling around the insertion site. b. The IVF has slowed or stopped. There is edema, pallor, coolness and firmness to touch at the insertion site. The client also c/o pain at the insertion site. c. The IVF has slowed or stopped. The client c/o burning and there is swelling and coolness at the insertion site. Blanching and blistering are also noted. d. The IVF has slowed or stopped. There is redness, pain, warmth, localized edema, and a palpable cord along the vein at the insertion site.

The IVF has slowed or stopped. There is edema, pallor, coolness and firmness to touch at the insertion site. The client also c/o pain at the insertion site.

A nurse is caring for a client who has a peripherally inserted central catheter (PICC). For which of the following complications should the nurse monitor? The need for multiple IV sticks Infection at the access site Dehydration Infiltration

The need for multiple IV sticksThe purpose of the PICC line is to help prevent the need for multiple peripheral IV sticks as it provides more reliable access that can be used long term. Infection at the access siteThis nurse should monitor the client who has a PICC for complications, such as infection at the access site and blood clots. It is important for the nurse to use aseptic technique when accessing and flushing the PICC line and during dressing changes. DehydrationPICC lines can be used to provide fluids for clients with dehydration when peripheral access is unattainable. InfiltrationMY ANSWERPICC lines have less incidence of infiltration due to the location of their insertion site. PICC lines are placed in large veins, such as the basilic or cephalic vein in the forearm or the antecubital fossa.

The nurse has just started a blood transfusion. Which statement is correct about the administration of blood? a. Vital signs will be obtained at 10 minutes, 20 minutes, 45 minutes, and then every 2 hours. b. Give blood slowly until the first 200 mL of blood has infused. c. After the unit has infused, close the roller clamp to the blood product and discard the unit of blood in the trash receptacle. d. The nurse will remain with the client during the first 5 minutes of initiating the transfusion.

The nurse will remain with the client during the first 5 minutes of initiating the transfusion.

A nurse is assessing a client who is restricted to bedrest for manifestations of right-sided heart failure. Which of the following findings should the nurse expect? Weak peripheral pulses Angina Sacrum edema Crackles in the lungs

Weak peripheral pulse A client who has left-sided heart failure will have manifestations of weak peripheral pulses due to decreased cardiac output. AnginaA client who has left-sided heart failure will have manifestations of angina due to decreased cardiac output. C;Sacrum edemaA client who has right-sided heart failure and is restricted to bedrest will often have fluid accumulation in the sacral area when compared to a client who is ambulatory. The edema is caused by increased systemic venous pressures and congestion. Crackles in the lungsA client who has left-sided heart failure will have manifestations of crackles in the lungs due to decreased cardiac output.

Prior to transfusing blood to a client, TWO registered nurses must verify which of the following? Select all that apply. a. Check the correct IV (intravenous) size and location. b. Check the compatibility of the patient and the donor blood group and the RH type. c. Check the unit number of the blood. d. Check the expiration date of the blood. e. Check the bag for coloring or clots.

b. Check the compatibility of the patient and the donor blood group and the RH type. c. Check the unit number of the blood. d. Check the expiration date of the blood. e. Check the bag for coloring or clots.

How may you assure client safety during a transfusion?

You should follow verification processes properly using orders, proper identification, and a second nurse check. Taking an initial set of vital signs prior to infusion is important for establishing the patient's baseline and monitoring for abnormalities. Nurses shoule be monitoring patients for 15-30 minutes after transfusion begins and obtaining vital per facility policy to ensure reactions don't occur, such as rash, itching, hypotension, tachycardia, and various other abnormalities, and to respond timely if they do. Patient awareness and education can also be a key component for quick medical action if reactions occur; therefore, instructing the patient and family on possible problems and encouraging them to notify you of any unusual changes during the transfusion is very important

A nurse is caring for a client who has a central venous catheter. When flushing the catheter, the nurse should use a 10-mL syringe to prevent which of the following complications associated with central vascular access devices? Catheter rupture Catheter migration Pneumothorax Phlebitis

correct; Catheter ruptureWhen injecting fluid through a catheter, a smaller syringe generates more pressure than a larger syringe does. Therefore, to reduce the risk of catheter rupture, syringes that are 10-mL or larger are recommended for flushing or injecting fluid into a central venous catheter. Catheter migrationUsing a 10-mL syringe to flush a central venous catheter will not prevent catheter migration. PneumothoraxUsing a 10-mL syringe to flush a central venous catheter will not prevent pneumothorax. PhlebitisUsing a 10-mL syringe to flush a central venous catheter will not prevent phlebitis.

A nurse is caring for a client who is about to receive a unit of packed RBCs and states, "This is my third unit of blood today. I don't want to get some disease from all this blood." Which of the following responses should the nurse make? "It is impossible for you to get an infection from donor blood." "Donated blood is carefully screened for infectious diseases." "The U.S. blood supply is among the safest in the world." "Why not ask your doctor about other treatment options?"

"It is impossible for you to get an infection from donor blood."This statement is not entirely true. Although the risk is low, disease transmission through blood transmission is possible. Improvements in blood-processing methods have reduced the number of infections resulting from the use of these blood products, but they have not eliminated them. Currently, the risk of blood-borne infection through receiving a blood transfusion or blood products in the U.S. is extremely low and has become progressively lower, but there is no absolute guarantee of safety. "Donated blood is carefully screened for infectious diseases."MY ANSWERThis statement is accurate. The nurse might continue to explain that the approach to blood safety in the U.S. includes stringent donor selection practices and the use of screening tests for HIV, AIDS, hepatitis B and C, syphilis, and other infectious diseases. Infected blood and blood products are safely discarded and are not used for transfusions. "The U.S. blood supply is among the safest in the world."Although this statement is true, it offers an empty type of reassurance without any solid information to address the client's fears. In fact, giving reassurance is a nontherapeutic communication technique. It implies that there is no need for concern and devalues the client's feelings, which might discourage them from further expression of their concerns. "Why not ask your doctor about other treatment options?"This is response is inappropriate not only because there might not be any other treatment options that offer comparable or better therapeutic benefits for this client, but by suggesting other options, the nurse is risking increasing the client's fears about transfusion. This response can sound as though the nurse is confirming that the client's concerns about infectious disease are entirely realistic.

Blood transfusion lab

1 Verify MD order, client ID and blood compatibility; verifying orders to patient identifiers is an immediate course of action prior to treating clients to ensure safe care. Blood compatibility checking is important to verify prior to retrieving blood from the blood bank to limit errors and setbacks as orders cannot be fulfilled without this step. - 2 Start IV; Ensuring the patient has a patent I.V. line that's functioning properly is important prior to obtaining blood as only 30 minutes is allotted for the transfusion to begin once the blood is on the unit. When using a peripheral line, a 20-22 gauge needle is needed as any size smaller could cause occlusions or slow the infusion. A Y-type blood administration set and 0.9% sodium chloride solution is used for the transfusion and should be ready prior to obtaining blood. - 3 Take the initial set of vital signs; it is important to ensure the client does not have an elevated temperature of 100.7 or more prior to obtaining blood as transfusion will be postponed until fever breaks and/or the provider modifies orders. This being the case, this should be the action you take right before going to the blood bank. Also, assessing the patient by obtaining and documenting a pretransfusion full set of vital signs is crucial to determine a baseline for the client in case a reaction occurs during transfusion. - 4 Pick up blood from the lab; after ensuring the patient and supplies are ready for the blood, you are ready to obtain the blood. Again, blood must be administered within 30 minutes of it entering the floor so this would be the step taken right before starting the transfusion. - 5 Start transfusion; blood cannot be administered until all of the following steps are properly taken prior as it is not safe.

A nurse is caring for a client who requires long-term central venous access and is an avid swimmer. Which of the following central venous access devices is the best choice for this client? A tunneled central catheter An implanted port A nontunneled percutaneous central catheter A peripherally inserted central catheter

A tunneled central catheterAlthough it is possible for a client who has a tunneled central catheter to swim, many providers discourage it due to the increased risk for infection via the exit site. It is also inconvenient to have to cover the site with plastic to protect the site and its dressing. An implanted portMY ANSWERBecause the entire device lies beneath the skin, the client can be immersed in water when the device is not in use without any increased risk for infection. This is the best choice for clients who wish to continue aquatic activities. A nontunneled percutaneous central catheterAlthough it is possible for a client who has a nontunneled percutaneous central catheter to swim, many providers discourage it due to the increased risk for infection via the exit site. It is also inconvenient to have to cover the site with plastic to protect the site and its dressing. A peripherally inserted central catheterAlthough it is possible for a client who has a peripherally inserted central catheter to swim, many providers discourage it due to the increased risk for infection via the exit site. It is also inconvenient to have to cover the site with plastic to protect the site and its dressing.

A nurse in the emergency department is caring for a client who was in a motor-vehicle crash. The provider determines that the client needs immediate central venous access for fluid and blood replacement. Which of the following central venous access devices should the nurse anticipate being inserted? A tunneled central catheter An implanted port A nontunneled percutaneous central catheter A peripherally inserted central catheter

A tunneled central catheterThis type of central catheter is designed for long-term therapy, such as lengthy courses of chemotherapy or parenteral nutrition. There is no indication that this client will require prolonged central venous access. Also, this type of device is not typically inserted quickly in an emergency setting. An implanted portThis type of central catheter is designed for long-term therapy, such as lengthy courses of chemotherapy or parenteral nutrition. There is no indication that this client will require prolonged central venous access. Also, this type of device is not typically inserted quickly in an emergency setting. correct; A nontunneled percutaneous central catheterThis type of central catheter is ideal for emergency situations where short-term (less than 6 weeks) central venous access is required for multiple therapies. This is the appropriate choice for this client. A peripherally inserted central catheterThis type of central catheter is designed for longer term therapy, such as lengthy courses of chemotherapy or parenteral nutrition. There is no indication that this client will require prolonged central venous access.

A nurse is participating in a blood drive and is taking a donation from a client who has type A- blood. The client asks the nurse what blood types can receive their blood donation. Which of the following responses should the nurse make? (Select all that apply.) A+ B+ O+ AB- AB+ A-

A+ is correct. Clients who have A+ blood type can receive blood from donors who have A- blood type. B+ is incorrect. Clients who have B+ blood type cannot receive blood from donors who have A- blood type. O+ is incorrect. Clients who have O+ blood type cannot receive blood from donors who have A- blood type. AB- is correct. Clients who have AB- blood type can receive blood from donors who have A- blood type. AB+ is correct. Clients who have AB+ blood type can receive blood from donors who have A- blood type. A- is correct. Clients who have A- blood type can receive blood from donors who have A- blood type.

A nurse is teaching a newly licensed nurse about using the nursing process when caring for a client who has an acid-base imbalance. The nurse should include that the stages of the nursing process should be performed in what order? (Move the steps into the box on the right, placing them in the order of performance. Use all the steps.) Assessment Implementation Analysis Planning Evaluation

Assessment is the first step. Assessment is the first step and includes gathering subjective and objective data, including, but not limited to, vital signs and lab and blood gas reports.Analysis is the second step. Analysis is the second step and includes examining and interpreting all the subjective and objective information collected during the first step.Planning is the third step. Planning is the third step and includes having the nurse work collaboratively with the client in the establishment of client centered goals and the development of a plan of care for the client.Implementation is the fourth step. Implementation is the fourth step and involves the nurse implementing interventions for the client that were developed in the planning stage to assist the client with meeting their goals.Evaluation is the final step. It is at this time that the nurse evaluates the implementation of the interventions provided for the client to determine if they are effective and if the goals of the plan of care are attainable.

A nurse is reviewing laboratory values for a client who has chronic kidney failure (CKF). Which of the following values should the nurse expect?

BUN 18 mg/dL Clients who have CKF will have an increased BUN level caused by the inability of the kidneys to excrete waste and the buildup of nitrogenous products. This BUN level is within the expected reference range of 10 to 20 mg/dL. C;Potassium 5.6 mEq/LThe most common cause of hyperkalemia is acute or chronic kidney failure because the damaged kidneys are unable to excrete potassium. This potassium level is above the expected reference range of 3.5 to 5 mEq/L. Hematocrit 40%Clients who have CKF will have a decreased hematocrit level due to anemia caused by a reduced production of erythropoietin by the kidneys. This hematocrit level is within the expected reference range of 37% to 52%. Creatinine 0.9 mg/dLClients who have CKF will have an increased creatinine level caused by the inability of the kidneys to excrete waste and the buildup of nitrogenous products. This creatinine level is within the expected reference range of 0.5 to 1.3 mg/dL.

A nurse is assessing a client who has chronic kidney disease (CKD). Which of the following findings should the nurse identify as a manifestation of the early stages of CKD? Polyuria Hypotension Increased appetite Jaundice skin tone

C; PolyuriaIn the early stages of CKD, tubular reabsorption of water is reduced and the urine becomes more dilute and clear. This causes the client to experience polyuria nocturia, which progresses to oliguria in the later stages of CKD. Hypotension \Hypertension is an expected finding in a client who has CKD. Hypertension can be a causative factor or the result of CKD. As the disease progresses, fluid overload and hypernatremia lead to an increase in the client's blood pressure. Increased appetiteAnorexia is an expected finding in a client who has CKD and is more common in clients who are in the later stages as uremia develops. Jaundice skin toneIn the later stages of CKD, the client can develop a uremic skin color of yellow-green for light-skinned clients, or a deeper darkening of the skin for clients who have darker skin tones. The skin can also become dry skin and develop uremic frost.

A nurse is caring for a client who has metabolic alkalosis. Which of the following findings should the nurse identify as a compensation mechanism for metabolic alkalosis? Hypoventilation Hyperventilation Increased renal acid excretion Decreased renal acid excretion

C;HypoventilationHypoventilation is the mechanism that helps clients compensate for metabolic alkalosis. As a result, the client's PaCO2 and HCO3- will increase. HyperventilationHyperventilation is the mechanism that helps clients compensate for metabolic acidosis. Increased renal acid excretionIncreased renal acid excretion is the mechanism that helps clients compensate for respiratory acidosis. Decreased renal acid excretionDecreased renal acid excretion is the mechanism that helps clients compensate for respiratory alkalosis.

A nurse is caring for a client who is receiving 0.9% sodium chloride IV at 75 mL/hr through a triple lumen central venous access device. The IV pump alarm sounds, indicating that there is an occlusion. Which of the following actions should the nurse take first? Call the provider who inserted the catheter. Flush the line with a 10-mL syringe of heparin. Check the line at or above the hub for kinked tubing that is creating a resistance to flow. Reposition the client.

Call the provider who inserted the catheter.The nurse should call the provider who inserted the catheter to report the occlusion. However, there is another action the nurse should take first. Flush the line with a 10-mL syringe of heparin.The nurse should flush the line with a 10-mL syringe of heparin, following agency policy, to liquify the occlusion. However, there is another action the nurse should take first. Check the line at or above the hub for kinked tubing that is creating a resistance to flow.MY ANSWERThe first action the nurse should take when using the nursing process is to assess the client's IV line at or above the hub for kinked tubing that is creating a resistance to the flow of the infusion. This is most likely the problem and should be where the nurse checks first. Reposition the client.The nurse should reposition the client to determine if the line is kinked or occluded by the client resting on the tubing. However, there is another action the nurse should take first.

A nurse is caring for an older adult client who is experiencing dehydration. The nurse should identify that which of the following factors increases the risk for dehydration in older adult clients? (Select all that apply.) Decreased kidney function Decreased thirst response Decreased total body fluid Eating watermelon daily Eating cucumbers with each meal

Decreased kidney function is correct. The nurse should identify that an older adult client who has decreased kidney function is at an increased risk for dehydration. Decreased thirst response is correct. The nurse should expect that an older adult client who has a decreased thirst response might not drink enough fluids daily, which puts them at increased risk for dehydration. Decreased total body fluid is correct. The nurse should expect that an older adult client who has decreased total body fluid to be at increased risk for dehydration. Eating watermelon daily is incorrect. Foods with high water content such as watermelon help rehydrate the client and decrease the risk for dehydration. Eating cucumbers with each meal is incorrect. Foods with high water content such as cucumbers help rehydrate the client and decrease the risk for dehydration.

A nurse caring for a client who is experiencing hypovolemia. Which of the following findings should the nurse identify as the priority to report to the provider? Dry mucous membranes Decreased urine output Report of thirst Decrease in level of consciousness

Dry mucous membranesDry mucous membranes are non-urgent because it is an early and expected finding of hypovolemia. Decreased urine outputDecreased urine output is non-urgent because it is an early and expected finding of hypovolemia. Report of thirstReport of thirst is non-urgent because it is an early and expected finding of hypovolemia. Decrease in level of consciousnessMY ANSWERWhen using the urgent vs non-urgent priority framework, the nurse determines that the priority finding is a decrease in the client's level of consciousness. This is an indication that the hypovolemia has progressed to a critical level and requires immediate intervention.

Which of the following actions should a nurse take prior to starting a blood transfusion? Establish intravenous access with a 22-gauge needle. Prime an infusion set with lactated Ringer's solution. Ensure that informed consent has been obtained from the client. Suggest that the client consider autologous transfusion.

Establish intravenous access with a 22-gauge needle.Blood is too viscous to infuse properly through a needle with a lumen this narrow. It is best to use an 18- or 19-gauge needle. Prime an infusion set with lactated Ringer's solution.Blood is always administered with 0.9% sodium chloride solution, and never with a solution that contains dextrose, because it causes lysis of red blood cells, or with lactated Ringer's. Ensure that informed consent has been obtained from the client.MY ANSWERIt is the responsibility of the prescribing health care provider to answer the client's questions about the need, risks, and benefits of a procedure. A nurse can witness the client's signature indicating informed consent. This must be done prior to obtaining or administering the blood. Suggest that the client consider autologous transfusion.Autologous transfusion is the process of collecting, storing, and reinfusing the client's own blood. It is an alternative for clients who choose not to receive blood components from recruited blood donors, but it is not possible in all situations. Typically it must be planned in advance and some clients are not eligible for it. There would be no need to suggest an autologous transfusion to a client who has no objection to a standard blood transfusion.

A nurse has completed assessing and analyzing data for a client who has an acid-base imbalance. Which of the following steps of the nursing process should the nurse take next? Implementation Reassessment Evaluation Planning

ImplementationThe nurse should move into the implementation phase of the nursing process following the planning stage. The plan of care will be carried out within the implementation stage. ReassessmentReassessment should occur if there is a change in the client's condition. If no change is evident, the nurse should continue to move into the next stage of the nursing process. EvaluationThe evaluation stage is the final stage of the nursing process. It takes place following the implementation of the plan of care. PlanningMY ANSWERAfter assessing and analyzing data for a client who has an acid-base imbalance, the nurse should move into the planning stage of the nursing process and establish goals and outcomes for the client.

A nurse is caring for a client who is experiencing respiratory alkalosis. Which of the following actions should be the goal of treatment for the client? Increase the carbon dioxide level. Increase the respiratory rate. Increase the bicarbonate level. Increase the pH level.

Increase the carbon dioxide level.MY ANSWERA state of respiratory alkalosis indicates that the client's carbon dioxide level is currently below the expected reference range. The goal of treatment should be to raise the level of carbon dioxide level back to within the expected reference range for PaCO2 of 35 to 45 mm Hg. Increase the respiratory rate.A state of respiratory alkalosis indicates the client is likely experiencing an elevated respiratory rate. Hyperventilation can cause respiratory alkalosis because carbon dioxide is eliminated as expirations increase. The goal of treatment should be to lower the client's respiratory rate. Increase the bicarbonate level.A state of respiratory alkalosis indicates that the client's bicarbonate level is currently above the expected reference range. The goal of treatment should be to lower the level of bicarbonate back to within the expected reference range for HCO3- of 21 to 28 mEq/L. Increase the pH level.A state of respiratory alkalosis indicates that the client's pH level is currently above the expected reference range. The goal of treatment should be to lower the pH level to within the expected reference range of 7.35 to 7.45.

A nurse is planning care for a client who has heart failure. Which of the following interventions should the nurse include in the plan of care? Maintain sodium intake at 3.5 g daily. Limit fluid intake to 2 L per day. Administer NSAIDs for minor discomfort. Place the client in a lateral recumbent position.

Maintain sodium intake at 3.5 g daily.The nurse should plan to limit the client's sodium intake to between 2 to 3 g daily to decrease fluid retention, which decreases the workload of the heart. C;Limit fluid intake to 2 L per day.The nurse should plan to limit the client's fluid intake to 2 L daily to decrease fluid retention, which decreases the workload of the heart. Administer NSAIDs for minor discomfort.The nurse should avoid administering NSAIDs to prevent sodium and fluid retention. Place the client in a lateral recumbent position.The nurse should elevate the head of the client's bed 45° to promote expansion of the chest wall and facilitate breathing.

A nurse is reviewing laboratory results for a client and notes the following arterial blood gas (ABG) values: pH 7.31, PaCO2 49 mm Hg, and HCO3- 25 mEq/L. The nurse should interpret these findings as an indication of which of the following acid-base imbalances? Metabolic acidosis Respiratory acidosis Metabolic alkalosis Respiratory alkalosis

Metabolic acidosisMetabolic acidosis is indicated by a pH value below the expected reference range and an HCO3- value below the expected reference range. Respiratory acidosisMY ANSWERThis client's ABG values indicate respiratory acidosis. Respiratory acidosis is indicated by a pH value below the expected reference range of 7.35 to 7.45 and a PaCO2 value above the expected reference range of 35 to 45 mm Hg. Metabolic alkalosisMetabolic alkalosis is indicated by a pH value above the expected reference range and an HCO3- value above the expected reference range. Respiratory alkalosisRespiratory alkalosis is indicated by a pH value above the expected reference range and a PaCO2 value below the expected reference range.

A nurse is reviewing the arterial blood gas (ABG) values for a client and notes the following results: pH 7.49, PaCO2 39 mm Hg, and HCO3- 35 mEq/L. The nurse should interpret this ABG reading as an indication of which of the following acid-base imbalances? Metabolic acidosis Respiratory acidosis Metabolic alkalosis Respiratory alkalosis

Metabolic acidosisMetabolic acidosis is indicated by a pH value less than 7.35 and an HCO3- value less than 21 mEq/L. Respiratory acidosisRespiratory acidosis is indicated by a pH value less than 7.35 and a PaCO2 value greater than 45 mm Hg. Metabolic alkalosisMY ANSWERThis client's ABG values indicate metabolic alkalosis. Metabolic alkalosis is indicated by a pH value above the expected reference range of 7.35 to 7.45, an HCO3- value above the expected reference range of 21 to 28 mEq/L, and a PaCO2 level within the expected reference range of 35 to 45 mm Hg. Respiratory alkalosisRespiratory alkalosis is indicated by a pH value greater than 7.45 and a PaCO2 level less than 35 mm Hg.

. List 4 symptoms of a hemolytic transfusion reaction, the cause and what the nurse should do?

Nausea/vomiting, tachycardia, hypotension, and hematuria are symptoms of a hemolytic transfusion reaction. These reactions occur due to blood incompatibility due to certain blood types antibodies spiking an immune response in the recipient's body. Nurses should verify the blood type with doctor's orders and the patient's ID then have another nurse as well prior to transfusion. Once verified and the baseline vitals are taken, the nurse should stay with the patient for the first 15-30 minutes to ensure no undesired reactions occur. If you notice an adverse reaction, stop the blood immediately, disconnect the line from the patient and assess vitals to ensure the patient is stable by maintaining airway, breathing, circulation. Notify the healthcare provider as soon as possible and keep the line patent by hanging a new bag of 0.9% sodium chloride solution. Don't turn on the saline connected to the Y-set because this will cause the remaining blood in the tubing to go to your patient, which may exacerbate the problem. Don't discard the blood; wait for direction from the healthcare provider. If the decision is made to stop the transfusion, then you'll need to follow your facility's policy on returning the blood to the blood bank. Document situation per facility policy.

Central line change NN

Nurses Note for Central Line Dressing Change Date/Time Notes 1/11/2023; 0900 Sterile right subclavian central line dressing change as per order/policy. Explained procedure, masked patient. Patient verbalizes understanding. Old dressing intact and clean. Insertion site without redness, drainage, warmth or swelling. Distance from insertion site to hub 7 cm consistent with previous measurement. Patient tolerated procedure well. No c/o. Bed in low position and locked. Call light within reach. Teresa Blanc, SN CCCTI------------------------------------

What should the nurse do if the blood tag and the client information does not match?

Nurses should not hang the blood if any differences are found during any part of the verification. Stop and notify the blood bank

A nurse is caring for a client who has chronic kidney disease (CKD). Which of the following actions should the nurse take to manage fluid overload? Obtain the client's blood pressure at least every 4 hr. Weigh the client periodically throughout the day. Measure the client's output every 8 hr. Limit client's oral fluid intake to mealtimes.

Obtain the client's blood pressure at least every 4 hr.MY ANSWERThe nurse should assess the client's blood pressure at least every 4 hr. An increase in the client's blood pressure can indicate fluid overload and hypertension, which can lead to further kidney damage. Weigh the client periodically throughout the day.The nurse should weigh the client daily at the same time each day. The nurse should ensure the client is in the same type of clothing and that the same scales are used each time the client is weighed. Measure the client's output every 8 hr.The nurse should keep an accurate measurement of the client's intake and output by measuring the client's output at least every 4 hr. Limit client's oral fluid intake to mealtimes.For clients on daily fluid restrictions, the nurse should assist the client to spread their oral fluid intake over the entire 24 hr of each day. Medications should be administered with meals when possible to reduce total liquids consumed for this purpose.

A nurse on a pediatric floor is teaching a newly licensed nurse about IV therapy. Which of the following information should the nurse include? Perform range of motion exercises on the extremity containing the IV site. Shave the client's hair if the IV is to be placed in the scalp. IV sites can be placed in the lower extremities up to the age of 2 years. Monitor the IV site, tubing, and connections every 4 hr.

Perform range of motion exercises on the extremity containing the IV site.MY ANSWERThe nurse should instruct the newly licensed nurse to perform range of motion exercises on the client's extremity that contains the IV site. Shave the client's hair if the IV is to be placed in the scalp.The nurse should instruct the newly licensed nurse that if the area selected for the site has an excessive amount of hair, scissors or surgical clippers should be used to remove the hair. Areas of the body should not be shaved because this could cause microabrasions. IV sites can be placed in the lower extremities up to the age of 2 years.The nurse should instruct the newly licensed nurse that sites in the lower extremities are used only until the pediatric client can crawl or walk, which should occur prior to the age of 2 years. Monitor the IV site, tubing, and connections every 4 hr.The nurse should instruct the newly licensed nurse to monitor the IV site, tubing and connections, and flow rate every hour for a pediatric client.

A nurse is providing teaching to a client who has chronic kidney failure and is scheduled to begin hemodialysis. Which of the following access sites should the nurse describe to the client? Peripherally inserted central catheter (PICC) Central venous catheter Arteriovenous fistula (AVF) Peripheral IV catheter

Peripherally inserted central catheter (PICC)PICCs are not used for the purpose of hemodialysis. Central venous catheter Central venous catheters are not used for the purpose of hemodialysis. C;Arteriovenous fistula (AVF)AVFs are one of the internal accesses used for hemodialysis therapy. An AVF is created by surgically joining an artery and a vein. Peripheral IV catheterPeripheral IV catheters are not used for the purpose of hemodialysis.

A nurse is assessing a client who is receiving 0.9% sodium chloride IV at 125 mL/hr. Which of the following should the nurse recognize as a possible complication related to the IV therapy? Petechiae is present over the IV site. The skin is cool over the IV site. Client reports coughing and shortness of breath. Client's blood pressure is lower than normal.

Petechiae is present over the IV site.Petechiae can indicate capillary fragility, but this finding is not a complication of IV therapy. The skin is cool over the IV site.The skin is generally cool over the IV site because the fluids that are being infused are at room temperature, and the client's body temperature is higher than the room temperature. Client reports coughing and shortness of breath.MY ANSWERCoughing and shortness of breath are manifestations of fluid overload. The nurse should slow the IV and notify the provider. Client's blood pressure is lower than normal.This should not be a complication of IV therapy. Higher blood pressure outside the expected reference range can indicate fluid overload.

A nurse is assessing a client who has been receiving IV therapy for several days and notes that the client's daily weight has increased. The nurse should identify that the client is at increased risk for developing which of the following IV-related complications? Phlebitis Extravasation Air embolism Circulatory overload

PhlebitisPhlebitis is a localized complication of IV therapy that occurs with inflammation of the inner lining of the vein. Indications of phlebitis include pain at the IV insertion site and surrounding area, swelling, erythema, and the presence of a palpable cord along the vein, but not weight gain. ExtravasationExtravasation is a localized complication of IV therapy that occurs when a vesicant medication or fluid moves from the vein into the surrounding tissue. Indications of extravasation include cool skin surrounding the IV site, localized edema, and report of pain from the client, but not weight gain. Air embolismAn air embolism is a medical emergency that occurs when a vessel such as the lung is obstructed by air. Indications of an air embolism include sudden difficulty in breathing, coughing, wheezing, hypotension, tachycardia, and chest or shoulder pain, but not weight gain. Circulatory overloadMY ANSWERThe nurse should identify that a client who has been receiving IV therapy and whose daily weight has increased is at risk for circulatory overload. The nurse should assess the client for other indications of circulatory overload, including tachycardia, increased blood pressure, edema, cough, and tachypnea. The nurse should also inform the provider of the client's increased weight.

A nurse is caring for a client who was admitted to the hospital for same day surgery and has a new prescription for continuous IV therapy. Which of the following actions should the nurse take when administering the IV therapy? Place a cold compress over the vein. Inspect the IV solution for fluid color, clarity, and expiration date. Apply a tourniquet 1 to 2 inches above the selected insertion site. Secure an armboard to the client's extremity.

Place a cold compress over the vein.Cold compresses will cause vasoconstriction and make the vein less visible. Inspect the IV solution for fluid color, clarity, and expiration date.MY ANSWERAll IV solutions must be free of contaminants and particles and current for usage. Apply a tourniquet 1 to 2 inches above the selected insertion site.A tourniquet should be applied 4 to 6 inches above the selected insertion site. Secure an armboard to the client's extremity.Armboards are applied to the client's extremity after the IV is started and only if necessary.

A nurse is reviewing the latest arterial blood gas (ABG) values for a client who is experiencing metabolic alkalosis. The nurse should identify that this action is part of which of the following steps of the nursing process? Planning Assessment Evaluation Implementation

PlanningThe planning stage of the nursing process includes developing goals and actions to provide care for a client. Reviewing the client's latest ABG values is not part of the planning stage. AssessmentThe assessment stage of the nursing process includes collecting subjective and objective data from the client to include when developing a plan of care. Reviewing the client's latest ABG values is not part of the assessment stage. EvaluationMY ANSWERReviewing the client's ABG values is part of the evaluation stage of the nursing process. During the evaluation stage, the nurse should determine if the actions taken in the implementation stage were successful in meeting the goals in the client's plan of care. ImplementationThe implementation stage of the nursing process includes providing nursing interventions that put the goals of the planning stage into action. Reviewing the client's latest ABG values is not part of the implementation stage.

A nurse is caring for a client who has a high fever and is hyperventilating. The client's ABG results are pH 7.51, PaCO2 28 mm Hg, and HCO3- 24 mEq/L. The nurse should identify that the client has which of the following acid-base imbalances? Respiratory acidosis Respiratory alkalosis Metabolic acidosis Metabolic alkalosis

Respiratory acidosisRespiratory acidosis is caused by inadequate excretion of carbon dioxide, which increases the hydrogen ion level of the blood, thereby causing the pH to decrease. A common cause of this acid-base imbalance is respiratory depression. Respiratory alkalosisMY ANSWERHyperventilation can cause clients to exhale excessive amounts of carbon dioxide. The loss of carbon dioxide decreases the hydrogen ion level of the blood, causing the pH to increase and resulting in respiratory alkalosis. Metabolic acidosisMetabolic acidosis results from a metabolic disturbance, such as diabetic ketoacidosis, seizures, or starvation, rather than a respiratory problem. Metabolic alkalosisMetabolic alkalosis results from a metabolic disturbance, such as prolonged vomiting, excessive antacid use, or hyperaldosteronism, rather than a respiratory problem.

A nurse is caring for a client who has a prescription to receive one unit of packed red blood cells. The client's blood type is AB+, and the nurse receives a unit of A- blood from the blood bank. Which of the following actions should the nurse take? Return the blood unit as it is not compatible with the client's blood type. Stay with the client for 15 min prior to starting the blood transfusion. Verify the unit of blood with another nurse. Prime the blood tubing with 0.45% sodium chloride.

Return the blood unit as it is not compatible with the client's blood type.A client with AB+ blood type can receive A- blood as the blood types are compatible. Clients who have type AB+ are universal receivers, which means they can receive blood from any donor blood of any blood type. Stay with the client for 15 min prior to starting the blood transfusion.The nurse should remain with the client for the first 15 min after the transfusion has started to make sure the client does not experience a transfusion reaction. Verify the unit of blood with another nurse.MY ANSWERThe unit of blood is compatible with the client's blood type. However, the nurse should ensure that the blood unit had been verified by two nurses before initiating the transfusion. Prime the blood tubing with 0.45% sodium chloride.The nurse should identify that 0.9% sodium chloride is the only IV solution that should be used to prime the blood tubing.

A nurse is caring for a client who has a central venous catheter and suddenly develops dyspnea, tachycardia, and dizziness. The nurse suspects an air embolism and clamps the catheter immediately. The nurse should reposition the client into which of the following positions? Supine with a pillow beneath the knees On their left side in Trendelenburg position Upright and leaning over the overbed table On their right side with the head of the bed elevated 15°

Supine with a pillow beneath the kneesThis is not the appropriate position when an air embolism is suspected. On their left side in Trendelenburg positionMY ANSWERThis position helps trap the air in the apex of the right atrium rather than allowing it to enter the right ventricle and, from there, move to the pulmonary arterial system. Upright and leaning over the overbed tableThis is not the appropriate position when an air embolism is suspected. On their right side with the head of the bed elevated 15°This is not the appropriate position when an air embolism is suspected.

A nurse is assessing a client who is exhibiting signs of a fluid and electrolyte imbalance. Which of the following findings should the nurse identify as a potential cause for the client's fluid and electrolyte imbalance? The client reports working in a warehouse in 21.1° C (70° F) temperature. The client reports that they performed yard work for 8 hr in 35° C (95° F) temperature earlier that day. The client reports that their provider decreased their diuretic dose. The client reports they had a 24-hr intestinal virus 2 weeks ago.

The client reports working in a warehouse in 21.1° C (70° F) temperature.The client is not at risk for excessive sweating that could lead to a fluid and electrolyte imbalance while working in a controlled temperature of 21.1° C (70° F). The client reports that they performed yard work for 8 hr in 35° C (95° F) temperature earlier that day.MY ANSWERThe nurse should identify that working outside in high temperatures for an extended period can cause profuse sweating and lead to a fluid and electrolyte imbalance. The client reports that their provider decreased their diuretic dose.When a diuretic dose is decreased, the client will experience a decrease in urination. This should not cause a fluid and electrolyte imbalance. The client reports they had a 24-hr intestinal virus 2 weeks ago.Although an intestinal virus increases the client's risk for a fluid and electrolyte imbalance during infection, this would not cause a fluid and electrolyte balance after 2 weeks.

A nurse is caring for a client who is receiving treatment for hyponatremia. The nurse should identify that which of the following findings is an indication that the treatment has been effective? (Select all that apply.) The client states their muscle spasms are absent. The client reports a headache. The client denies being confused. The client reports being nauseated. The client reports feeling tired.

The client states their muscle spasms are absent is correct. The absence of muscle spasms indicates that the treatment for hyponatremia has been effective. The client reports a headache is incorrect. Headache is a manifestation of hyponatremia, rather than an indication that the treatment for hyponatremia has been effective. The client denies being confused is correct. The client reporting that they are not confused is an indication of adequate sodium levels. Therefore, this is an indication that the treatment for hyponatremia has been effective. The client reports being nauseated is incorrect. Nausea is a manifestation of hyponatremia, rather than an indication that the treatment for hyponatremia has been effective. The client reports feeling tired is incorrect. Fatigue is a manifestation of hyponatremia, rather than an indication that the treatment for hyponatremia has been effective.

A charge nurse is observing a newly licensed nurse who is preparing to administer a blood transfusion to a client. For which of the following actions by the newly licensed nurse should the charge nurse intervene? The nurse selects 0.45% sodium chloride to use to prime the tubing. The nurse asks another nurse to check the blood unit label and client identification prior to beginning the transfusion. The nurse uses tubing with a filter for the blood transfusion. The nurse discards the tubing after the first unit of blood is completed.

The nurse selects 0.45% sodium chloride to use to prime the tubing.MY ANSWERThe charge nurse should intervene if the newly licensed nurse selects 0.45% sodium chloride to prime the tubing. The nurse should identify that 0.9% sodium chloride is the only IV solution that should be used to prime the tubing for blood administration. The nurse asks another nurse to check the blood unit label and client identification prior to beginning the transfusion.A client who is receiving blood or blood products should always have two nurses check the blood unit label and client identification for comparison. The newly licensed nurse should take this action because it decreases the chance of error. The nurse uses tubing with a filter for the blood transfusion.The newly licensed nurse should use tubing with a filter during blood administration to remove any clots and platelet clumps that might be present in the unit of blood. The nurse discards the tubing after the first unit of blood is completed.The newly licensed nurse should discard the tubing after each unit of blood is administered, or every 4 hr, whichever comes first.

6. How should the nurse verify the blood?

The verification process has two steps; 1) both nurses should make sure the information found on the tag attached to the blood matches the information on the patient's wristband, which includes validating the patient's first and last name, type and crossmatch number, identification number, and date of birth 2) check the tag attached to the blood with the label affixed to the blood, which includes the unit number, blood group/Rh, blood product, and expiration date. If you verify with another nurse that everything matches then both of you should sign the appropriate areas to indicate positive verification.

A nurse is caring for a client who has an implanted port that needs to be accessed for an infusion. Which of the following actions should the nurse take? Use a standard medium-gauge needle to access the port. Insert the primed needle into the port at a 45° angle. Withdraw the needle after insertion, leaving the needle's sheath in place for the infusion. Cover the device and the needle with a sterile transparent dressing.

Use a standard medium-gauge needle to access the port.An implanted port must be accessed with a non-coring needle. These needles have a deflected point that helps to avoid septal injury by slicing through the septum without coring out a tiny piece of it each time the port is accessed. Insert the primed needle into the port at a 45° angle.the primed needle should be inserted into the port at a 90° angle. Withdraw the needle after insertion, leaving the needle's sheath in place for the infusion.There is not a sheath covering the needle used to access an implanted port. correct Cover the device and the needle with a sterile transparent dressing.Once the implanted port has been accessed, the needle must be supported and anchored. The needle should be covered with a transparent dressing to secure the needle.

A nurse is preparing to administer an IV medication to a client. The nurse should identify that which of the following is a disadvantage of administering IV medications? IV medications are irreversible. IV medications have a slow onset. IV medications bypass the liver. IV medications have less bioavailability.

V medications are irreversible.MY ANSWEROnce an IV medication has been injected, it cannot be retrieved. If the dose is excessive or the client is allergic, the consequences can be fatal. IV medications have a slow onset.One of the potential benefits of IV medications is the ability to be absorbed instantaneously, especially in an emergency. IV medications bypass the liver.This is an advantage of IV medications because, by bypassing the liver, there is no inactivation by liver enzymes. IV medications have less bioavailability.Medications that are given intravenously are 100% bioavailable since they are injected directly into the bloodstream.

A nurse is participating in a blood drive and is taking a donation from a client who has type O+ blood. The client asks the nurse what type of blood they can receive. Which of the following statements should the nurse make? "You can receive a blood donation from donors with type O- and type O+ blood." "You can receive a blood donation from donors with type B- and type A+ blood." "You can receive a blood donation from donors with type B- and type AB+ blood." "You can receive a blood donation from donors with type AB- and type A- blood."

You can receive a blood donation from donors with type O- and type O+ blood."MY ANSWERClients who have type O+ blood can only receive blood from donors who have type O- or type O+ blood. Clients who have type O- blood are universal donors, meaning that anyone can receive their blood, regardless of blood type. "You can receive a blood donation from donors with type B- and type A+ blood."Clients who have type O+ blood cannot receive blood from donors with type B- or type A+ blood. "You can receive a blood donation from donors with type B- and type AB+ blood."Clients who have type O+ blood cannot receive blood from donors with type B- or type AB+ blood. "You can receive a blood donation from donors with type AB- and type A- blood."Clients who have O+ blood cannot receive blood from donors with type AB- or type A- blood.

The student nurse is reviewing the physician orders. The nurse understands which of the following clients would benefit from continuous intravenous (IV) fluids? Select all that apply. a. The client is NPO. b. The client is very nauseated with frequent vomiting, and unable to eat or keep liquids down. c. The client has a nasogastric tube to low, intermittent wall suction post surgery. d. The client was brought to the emergency department via EMS. There is a large amoung of bleeding noted from a gash in the thigh. e. The client is post surgery on sips of clear liquids only.

a. The client is NPO. b. The client is very nauseated with frequent vomiting, and unable to eat or keep liquids down. c. The client has a nasogastric tube to low, intermittent wall suction post surgery. d. The client was brought to the emergency department via EMS. There is a large amoung of bleeding noted from a gash in the thigh. e. The client is post surgery on sips of clear liquids only.

When administering a transfusion of packed red blood cells, it is important to allow the blood to warm to room temperature for 1 hr. make sure the entire unit is transfused within 4 hr. begin the blood transfusion at a rate of 10 mL/hr. change the blood tubing after every unit infused.

allow the blood to warm to room temperature for 1 hr.Blood cannot be left in a room-temperature environment for more than 30 min prior to infusion. Red blood cells can break down and release potassium into the bloodstream, thus putting the client at risk for hyperkalemia. As blood components warm, the risk of bacterial growth increases. If the transfusion is delayed, the blood cannot be refrigerated on the nursing unit. It must be kept at a specific temperature in the blood bank. make sure the entire unit is transfused within 4 hr.MY ANSWERInfusion times that exceed 4 hr increase the risk for bacterial proliferation. Ideally, a unit of packed RBCs is infused within 2 hr. Clients who are at risk for fluid-volume excess will require slower rates of infusion; however, the entire transfusion must not exceed 4 hr. begin the blood transfusion at a rate of 10 mL/hr.The blood product should not be infused at a rate faster than 5 mL/min for the first 15 min. This is because many of the more serious transfusion reactions occur after only a small amount of blood has been transfused. A rate of 2 mL/hr is preferable initially. After the first 15 min, it is generally safe to increase the flow to the prescribed rate. change the blood tubing after every unit infused.This varies with agency policy, but generally for clients receiving multiple units of blood, the blood tubing is changed after every two units transfused to reduce the risk of bacterial contamination. Some agencies specify that four to six units can be transfused without changing the blood tubing.

A nurse is caring for a client who is receiving dextrose in 5% water with 20 mEq of potassium chloride at 75 mL/hr. The provider has prescribed 1 g ceftriaxone IV. When preparing to administer this medication by intermittent IV bolus, which of the following actions should the nurse take first? Obtain the client's vital signs. Determine the client's level of consciousness. Verify the medication's compatibility with the primary IV solution. Check the amount of IV solution in the primary bag.

btain the client's vital signs.The nurse should obtain the client's vitals for baseline data. However, there is another action the nurse should take first. Determine the client's level of consciousness.The nurse should determine the client's level of consciousness for baseline data. However, there is another action the nurse should take first. Verify the medication's compatibility with the primary IV solution.MY ANSWERThe greatest risk to this client is injury from precipitate in the IV solution. Therefore, the first action the nurse should take is to assess the medication's compatibility with the primary solution. If the medication is not compatible with the primary solution, a precipitate can form in the IV tubing, preventing medication administration. Check the amount of IV solution in the primary bag.The nurse should check the amount of IV solution in the primary bag to see how much solution is left to be infused. However, there is another action the nurse should take first.

A nurse is preparing a blood transfusion for a client who has type A blood. The nurse should know that the client can safely receive blood from blood group O because type O blood contains no A antigens. type A blood contains O antibodies. type O blood contains no A antibodies. type A blood contains O antigens.

correct; type O blood contains no A antigens.Type O blood contains no antigens at all, which is why clients who have type O blood are considered universal donors. Their blood can be transfused to anyone who has any ABO-related blood type without putting them at risk for an ABO incompatibility. It is the specific antigens in the transfused blood that can trigger hemolytic reactions. Because type O blood has no antigens, it is safe for this client and for any other client. type A blood contains O antibodies.There are no O antibodies in any type of blood, so this client would not be in any danger from receiving type O blood. type O blood contains no A antibodies.Type O blood contains A and B antibodies. However, those antibodies would not cause harm to a client who has type A blood. type A blood contains O antigens.There are no O antigens in type A blood or in any other type of blood. There are only antigens for A, B, or a combination of A and B in some blood types.

A nurse started a transfusion of packed RBCs for a client 1 hr ago. The client has suddenly developed shaking chills, muscle stiffness, and a temperature of 38.6° C (101.5° F). The client appears flushed and reports a headache and "nervousness." The nurse should identify that the client has most likely developed which of the following types of transfusion reaction? Septic Acute hemolytic Allergic Febrile nonhemolytic

epticAlthough a rapid onset of chills and fever is common with septic reactions, the other manifestations this client is experiencing are not. Hypotension is a classic symptom of a septic reaction and, in most cases, symptoms develop after the unit is transfused and sometimes not until several hours later. The usual cause of this type of reaction is contaminated blood products. Acute hemolyticWhile an acute hemolytic type of reaction is heralded by fever, chills, and anxiety, it is also indicated by nausea, chest tightness, dyspnea, tachycardia, tachypnea, and hypotension. This type of reaction is usually immediate. It begins after transfusion of as little as 10 mL of blood. The cause is incompatibility of the recipient's blood with that of the donor. Allergiclthough flushing is common with allergic reactions, the other manifestations this client is experiencing are not. Hives (urticaria), itching, and wheezing are common signs of an allergic reaction. Allergic reactions are thought to be a result of a sensitivity reaction to a plasma protein in the donor's blood. CORRECT Febrile nonhemolyticThis is the most common type of transfusion reaction. The characteristic fever usually develops within 2 hr after the transfusion is started. Other classic symptoms include chills, headache, flushing, anxiety, and muscle pain. This type of reaction is usually a result of sensitization to the plasma, platelets, or white blood cells. Although this type of reaction is not life-threatening, it can be frightening and uncomfortable for the client.

A platelet transfusion is indicated for a patient who has a systemic infection. has thrombocytopenia. is in hypovolemic shock. has hemolytic anemia.

has a systemic infection.There is no particular blood component that can be infused to combat a systemic infection. Anti-infective medications are prescribed for clients who have systemic infections. has thrombocytopenia.MY ANSWERA client who has thrombocytopenia has a low platelet count. When platelet counts drop below 20,000/mm3, a transfusion of platelets is generally indicated for the client. is in hypovolemic shock.Hypovolemic shock occurs when the circulating vascular volume is too low to support adequate tissue perfusion. To help expand blood volume and increase tissue perfusion plasma is the usual choice. For severe hemorrhage, whole blood is sometimes transfused. has hemolytic anemia.Various types of hemolytic anemia, including sickle-cell anemia and thalassemia, are treated with transfusions of red blood cells.

Blood transfusion articles

https://cccti.mrooms.net/pluginfile.php/2091786/mod_folder/content/0/Blood%20Administration%20Lab/Article%20Avoiding%20Bad%20Blood.pdf?forcedownload=1 https://cccti.mrooms.net/pluginfile.php/2091786/mod_folder/content/0/Blood%20Administration%20Lab/Article%20Keeping%20Patients%20Safe%20When%20Receiving%20Blood.pdf?forcedownload=1 https://cccti.mrooms.net/pluginfile.php/2091786/mod_folder/content/0/Blood%20Administration%20Lab/Blood%20administration.pdf?forcedownload=1 https://cccti.mrooms.net/pluginfile.php/2091786/mod_folder/content/0/Blood%20Administration%20Lab/Blood%20Transfusion%20Form.docx?forcedownload=1 https://cccti.mrooms.net/pluginfile.php/2091786/mod_folder/content/0/Blood%20Administration%20Lab/Revised%20Directions%20for%20Blood%20Administration.docx?forcedownload=1 https://cccti.mrooms.net/pluginfile.php/2091786/mod_folder/content/0/Blood%20Administration%20Lab/The%20Client%20Receiving%20a%20Blood%20Transfusion.docx?forcedownload=1 https://cccti.mrooms.net/pluginfile.php/2091786/mod_folder/content/0/NC%20BON%20Blood%20Infusion%20Therapy.pdf?forcedownload=1

A client who is anticipating total hip replacement is considering autologous transfusion. When teaching this client about autologous transfusion, it is important to emphasize that it reduces the risk of mismatched blood. a hemoglobin level above 9.5 mg/dL is required to receive this transfusion. there is no need to test the blood for infectious diseases. donations can be made every other day.

it reduces the risk of mismatched blood.MY ANSWERMismatched blood can cause an immune response to another person's antigens. Because the client is their own donor in an autologous transfusion, there is no risk of exposure to another person's antigens. a hemoglobin level above 9.5 mg/dL is required to receive this transfusion.A hemoglobin level of at least 11 g/dL is required for autologous donation. there is no need to test the blood for infectious diseases.Blood collected for autologous transfusion is tested for HIV and the hepatitis B virus. A client who has either of these viruses is not eligible for autologous transfusion. donations can be made every other day.Donations can be made every 3 to 4 days.

A nurse is assessing a client who has left-sided heart failure with decreased cardiac output. Which of the following manifestations should the nurse expect? Flushing of the skin on exertion Respiratory rate of 16/min Warm lower extremities Nocturia at night

lushing of the skin on exertion A client who has left-sided heart failure with decreased cardiac output will appear pale in color on exertion due to poor tissue perfusion. Respiratory rate of 16/minA client who has left-sided heart failure with decreased cardiac output will have tachypnea due to intra-alveolar fluid causing engorgement of pulmonary vessels. Warm lower extremitiesA client who has left-sided heart failure with decreased cardiac output will have cool extremities due to poor tissue perfusion. C;Nocturia at nightA client who has left-sided heart failure with decreased cardiac output can have oliguria during the day and nocturia at night due to poor tissue perfusion to the kidneys.

A nurse is preparing to initiate the transfusion of a unit of packed RBCs to a client. After the unit of blood has arrived, which of the following procedures will help the nurse protect the client against the possibility of a blood-group incompatibility? Comparing the ID numbers on the blood unit with those on the order form and the client's wristband Obtaining a blood sample from the client for typing and crossmatching Giving a prescribed pre-medication 30 min prior to starting the transfusion Observing the client for 15 to 30 min after the transfusion is initiated

omparing the ID numbers on the blood unit with those on the order form and the client's wristbandMY ANSWERBefore administering blood or blood products, the client's nurse and one other authorized individual are required to identify the client and compare the label on the blood product to the prescription in the medical record and the client's armband. These actions will verify that the correct client is receiving the correct blood and help prevent the possibility of a blood-group incompatibility. Obtaining a blood sample from the client for typing and crossmatchingAlthough it is true that a type and crossmatch is done to prevent blood-group incompatibilities, the blood bank cannot issue the blood component until the type and crossmatch results are obtained. Therefore, this is not what the nurse would do at this point to ensure the client's safety. Giving a prescribed pre-medication 30 min prior to starting the transfusionIf the client has a history of adverse reactions to blood products, the provider might prescribe a pre-medication such as acetaminophen or another antipyretic agent. Administering the medication 30 min prior to transfusion helps ensure effective results of the pre-medication. However, this is done to prevent febrile reactions, not blood-group incompatibilities. Observing the client for 15 to 30 min after the transfusion is initiatedThis is standard procedure, because most transfusion reactions, including those related to blood-type incompatibilities, manifest during the infusion of the first 50 to 100 mL of the blood product and possibly sooner. This procedure helps detect, not prevent, the possibility of a blood-group incompatibility.

A nurse is reviewing a client's latest arterial blood gas (ABG) report. Which of the following values should the nurse identify as the priority to report to the provider? pH 7.37 PaCO2 43 mm Hg HCO3- 27 mEq/L PaO2 76 mm Hg

pH 7.37The value of pH 7.37 is nonurgent because it is within the expected reference range of 7.35 to 7.45. Therefore, there is another finding that is the nurse's priority to report to the provider. PaCO2 43 mm HgThe value of PaCO2 43 mm Hg is nonurgent because it is within the expected reference range of 35 to 45 mm Hg. Therefore, there is another finding that is the nurse's priority to report to the provider. HCO3- 27 mEq/LThe value of HCO3- 27 mEq/L is nonurgent because it is within the expected reference range of 21 to 28 mEq/L. Therefore, there is another finding that is the nurse's priority to report to the provider. PaO2 76 mm HgMY ANSWERWhen using the urgent vs. nonurgent approach to client care, the nurse should determine that the priority value to report to the provider is PaO2 76 mm Hg. This value is below the expected reference range of 80 to 100 mm Hg and could be an indication the client is decompensating.


संबंधित स्टडी सेट्स

Business Law: Chapter 1- Legal and Constitutional Foundations of Business

View Set

Computer Software & Hardware Architecture CIS

View Set

Healthcare Management Student Made Quizzes

View Set

AP Biology: Chapter 18: Regulation of Gene Expression

View Set

Chapter 5 - Adaptations to Anaerobic Training Programs

View Set

Medical Terminology Chapter 2: Congenital diseases

View Set

Place Value/ Standard and Expanded Form

View Set

MDT Engines Chapter 4 review questions

View Set